Download as pdf or txt
Download as pdf or txt
You are on page 1of 9

Andrew van Herick

Math 710
Dr. Alex Schuster
Nov. 2, 2005

Assignment 8
Section 3: 20, 24, 25, 30, 31

20. Show that the sum and product of two simple functions are simple. Show that

A\B = A B

A[B = A + B A B

Ac = 1 A

Proof.

Let E R and suppose f; g : E ! R are two simple functions. By Proposition 19 (p.


67, Royden) both f g and f + g are measurable. Because f and g are simple f (E) and
g (E) are …nite sets. It follows that both

fuv 2 R : u 2 f (E) ; v 2 g (E)g and


fu + v 2 R : u 2 f (E) ; v 2 g (E)g

each have cardinality which is at most jf (E)j jg (E)j ; in other words, …nite. It is easy to
see that

(f g) (E) fuv 2 R : u 2 f (E) ; v 2 g (E)g and


(f + g) (E) fu + v 2 R : u 2 f (E) ; v 2 g (E)g

Hence it follows that (f g) (E) and (f + g) (E) must be …nite as well. By de…nition
f g; f + g are simple functions.

Let A; B R:

First we’ll show that A\B = A B: Let x 2 R: If x 2 A \ B then x 2 A and x 2 B;


so A (x) = 1 and B = 1: Hence

A\B (x) = 1 = 1 1 = A (x) B (x) = A B (x) :

If x 2
= A \ B; then A\B = 0: By de Morgan’s laws x 2 Ac [ B c ; i.e. x 2 = A or x 2 = B:
Either way A (x) B (x) = 0. Hence A\B (x) = A B (x) : This covers all cases of x
and establishes that A\B (x) = A B (x) for all x 2 R: By de…nition A\B = A B:
2

Now we’ll show that Ac =1 A: Let y 2 R: If x 2 A then Ac (y) = 0 and A (y) =1


by de…nition, so
Ac (y) = 0 = 1 1=1 A (y) = (1 A ) (y) :

If x 2 Ac ; then then Ac (y) = 1 and A (y) = 0; so

Ac (y) = 1 = 1 0=1 A (y) = (1 A ) (y) :

This covers all cases of y and establishes that Ac (y) = (1 A ) (y) for all y 2 R: Hence
Ac = 1 A:

To show that A[B = A B. By de Morgan’s laws A [ B = Ac \ B c : By what we


proved above

A[B 1 = (A[B)c

= Ac \B c

= Ac Bc

= ( 1+ A ) (1 B)

= A B + B + A 1

It follows immediately that A[B = A + B A B; as desired.


Andrew van Herick 3

24. Let f be measurable and B a Borel set. Then f 1 [B] is a measurable set.

Proof.

Let f be measurable and B a Borel set. Let E := E R:f 1 [E] is measurable : We’ll
show that E contains B; the collection of Borel sets.

First we’ll show that E contains every open interval. Let I R be an open interval.
Then I = (a; b) for some (possibly extended) real numbers a; b such that a < b: Note that
I = ( 1; b) \ (a; 1) : As f is measurable f 1 (( 1; b)) = fx : f (x) < bg is measurable
by de…nition. Similarly f 1 ((a; 1)) is also measurable. By the fact that the measurable
sets form a -algebra

1 1 1 1
f (I) = f (( 1; b) \ (a; 1)) = f (( 1; b)) \ f ((a; 1))

is also measurable. Hence I 2 E: This establishes that I 2 E for every open interval I R:

Now we’ll show that E is a -algebra. Let A0 be a countable index set and fE g 2A0
be a collection of sets from E: Then by the fact that the measurable sets form a -algebra
0 1
[ [
f 1@ E A= f 1 (E )
2A0 2A0

is measurable, since f 1 (E ) 2A0 is a countable collection of measurable sets. Now let


c
E 2 E: By the same reasoning f 1 (E c ) is measurable since f 1 (E c ) = f 1 (E) : This
establishes that E is a -algebra.

Thus E is a -algebra containing every open interval. We proved in Exercise 10.4.5.


(p. 311, Wade) that every open set in R is a countable union of open intervals. It follows
immediately that every open subset of R belongs to E: This establishes that E is a -algebra
containing every open subset of R:

As B is the smallest -algebra containing all the open sets, it follows that B E: As
B 2 B; it follows immediately that B 2 E: Equivalently B is measurable.
4
Andrew van Herick 5

25. Show that if f is a measurable real valued function and g a continuous function de…ned
on ( 1; 1) ; then g f is measurable.

Proof.

Let f be a measurable real valued function and g a continuous function de…ned on


( 1; 1) : As of g f is R; an open set the domain of g f is measurable.

Let 2 R and E = fx : g f (x) > g : It is easy to see that

1
E = fx : g f (x) 2 (a; 1)g = (g f ) ((a; 1)) :

By Lemma 1 below E = f 1 g 1 (( ; 1)) : Because g : R ! R is continuous and ( ; 1)


is open, by Theorem 10.58 (p. 317, Wade) g 1 ((a; 1)) is is open in R: By de…nition
g 1 ((a; 1)) is a Borel set. Problem 24 above E = f 1 g 1 (( ; 1)) is a measurable set.

Lemma 1 Let E; F; G be sets and suppose f : E ! F and g : F ! G: It A G; then


(g f ) 1 (A) = f 1 g 1 (A) :

Proof. Let x 2 (g f ) 1 (A) : By de…nition (g f ) (x) = g (f (x)) 2 A: By the de…nition of


inverse images we get f (x) 2 g 1 (A) and x 2 f 1 g 1 (A) : Now let y 2 f 1 g 1 (A) :
Then f (y) 2 g 1 (A) : By de…nition g (f (y)) 2 A. As g (f (y)) = (g f ) (y) we have
(g f ) (y) 2 A: By de…nition y 2 (g f ) (A) : This establishes that (g f ) 1 (A) =
f 1 g 1 (A) :
6
Andrew van Herick 7

30. Prove Egoro¤’s Theorem: If hfn i is a sequence of measurable functions that converge
to a real-valued function f a.e. on a measurable set E of …nite measure, then given
> 0; there is a subset A E with mA < such that fn converges uniformly on
EnA: [Hint: Apply Proposition 24 repeatedly with "n = n1 and n = 2 n .]

Proof.

Let > 0: For each k 2 N de…ne "k = k1 ; k = 2 k . Clearly "k ; k > 0: Using
Proposition 24 (p. 73, Royden) for each k 2 N choose Ak E such that m (Ak ) < k , and
choose Nk 2 N such that for all x 2
= Ak and all n Nk ;

jfn (x) f (x)j < "k :


S
Let A := k2N Ak : Because each Ak E it follows that A E: Note that each Ak is
measurable. By Proposition 13 (p. 62, Royden)
1
X 1
X 1
X
k
m (A) = m (Ak ) < k = 2 =
k=1 k=1 k=1

This establishes that m (A) < :

Now we’ll show that fn converges uniformly on EnA: Let " > 0: Then there exists
1
M 2 N such that M < ": Choose N 2 N such that N = NM : Let n N and x 2 EnA:
T
As x 2 A and A = k2N (Ak )c ; we have x 2 (AM )c : From our construction of AM above
c c

1
jfn (x) f (x)j < "M = < ":
M
This establishes that for all " > 0 there is a N 2 N such that n N implies jfn (x) f (x)j <
" for all x 2 EnA: By de…nition fn is uniformly convergent on EnA:

This establishes the existence of a subset A E with mA < such that fn converges
uniformly on EnA:
8
Andrew van Herick 9

31. Prove Lusin’s Theorem: Let f be a measurable real-valued function on an interval


[a; b] : Then given > 0; there is a continuous function ' on [a; b] such that such that
m fx : f (x) 6= ' (x)g < : [Hint: Use Egoro¤’s Theorem, Propositions 15 and 22,
and Problem 2.40.]

Proof.

Let f be a measurable real-valued function on an interval [a; b] : Let > 0: As f is real


valued f takes on the values 1 on ;; a set of measure zero. Let k = k1 : By Proposition
22 (p. 71, Royden) for each k 2 N choose a continuous function hk on [a; b] such that
jf hk j < k : It follows almost immediately from this construction that hk (x) ! f (x)
for every x 2 [a; b] : By Egoro¤’s Theorem there is a subset A [a; b] with m (A) < 2
such that hk converges to f uniformly on [a; b] nA: By Proposition 15 (p. 63, Royden) there
exists a closed set F [a; b] nA such that m ([a; b] nAnF ) < 2 : Because all of the hk ’s are
continuous on [a; b] ; they are also continuous on F [a; b] : Hence f is continuous on F by
Exercise 7.7.1(a) (p. 191, Wade). By Problem 2.40 (p. 49, Royden) there is a continuous
function g : R ! R such that f (x) = g (x) for each x 2 F: Let ' = gj [a; b] : Then ' is
clearly continuous on [a; b] : Clearly fx 2 [a; b] : f (x) 6= ' (x)g ([a; b] nF ) [ A: Because
([a; b] nF ) [ A = ([a; b] nAnF ) [ A; it follows that

m (fx : f (x) 6= ' (x)g) m (([a; b] nF ) [ A) = m (([a; b] nAnF ) [ A)


< + =
2 2
This establishes the existence of a continuous function ' : [a; b] ! R such that m fx : f (x) 6= ' (x)g <
as desired.

You might also like